Find the missing length (picture below)

Find The Missing Length (picture Below)

Answers

Answer 1

Answer:

Step-by-step explanation:

because these are similar triangles, that is, one is a bigger of smaller version of the other, then we know, that the bigger triangle is just 2 times bigger than the smaller,  or   2x of any side of the small one

sooo   2(20) =40  

so we know that side n of the bigger triangle is 40


Related Questions

A professor is interested in whether or not college students have a preference (indicated by a satisfaction score) for reading a textbook that has a layout of one column or layout of two columns. In the above experiment, what is the dependent variable

Answers

Answer:

Satisfaction score

Step-by-step explanation:

The dependent variable may be described as the variable which is being measured in a research experiment. In the scenario described above, the dependent variable is the satisfaction score which is used to measure preference for a one or two column textbook. The dependent variable can also seen as the variable which we would like to predict, also called the predicted variable . The predicted variable here is the satisfaction score.

Please help me >_< will give out brainliest

Answers

Answer:  1080

====================================================

Explanation:

We have an octagon because there are n = 8 sides. The diagram below shows one way to number the sides so you can count them efficiently (without missing any or double counting any).

----------------

Plug n = 8 into the formula below

S = 180(n-2)

S = 180(8-2)

S = 180(6)

S = 1080

The 8 interior angles add up to 1080 degrees.

If the cutoff Z score on the comparison distribution is 2.33 and the sample value has a score of 2.35 on the comparison distribution, the correct decision is to:____.
A) fail to reject the null hypothesis.
B) reject the null hypothesis.
C) accept the researc hypothesis.
D) reject the research hypothesis.

Answers

Answer:

B) reject the null hypothesis.

Step-by-step explanation:

What does y equal in the solution of the system of equations below? 5y-3x-4z=22 2z-2x=-6 2z+3x=-6

Answers

9514 1404 393

Answer:

  y = 2

Step-by-step explanation:

Subtracting the second equation from the third gives ...

  (2z +3x) -(2z -2x) = (-6) -(-6)

  5x = 0

  x = 0

Using this in the third equation, we have ...

  2z +0 = -6

  z = -3

And substituting these values into the first equation, we have ...

  5y -3(0) -4(-3) = 22

  5y = 10 . . . . . subtract 12

  y = 2

__

The solution to the system is (x, y, z) = (0, 2, -3).

Which answers describe the shape below? Check all that apply.
A. Square
B. Quadrilateral
C. Rhombus
D. Trapezoid
E. Rectangle
F. Parallelogram

Answers

Answer:

b and f

Step-by-step explanation:

A 27​% solution ​( 27mg per 100 mL of​ solution) is given intravenously. Suppose a total of 1,36 L of the solution is given over a 10 ​-hour period. Complete parts​ (a) through​ (c) below.
a. What is the flow rate in units of​ mL/hr?
nothing ​mL/hr ​(Type an integer or decimal rounded to the nearest thousandth as​ needed.)
What is the flow rate in per​ hour?
nothing ​mg/hr ​(Type an integer or decimal rounded to the nearest thousandth as​ needed.)
b. If each mL contains 13 drops​ (the drop factor is expressed as ​gtt/mL), what is the flow rate in units of​ 13gtt/hr?
nothing ​gtt/hr ​(Type an integer or decimal rounded to the nearest thousandth as​ needed.)
c. During the 10 ​-hour ​period, how much is​ delivered?
nothing mg ​(Type an integer or decimal rounded to the nearest thousandth as​ needed.)

Answers

Answer:

Step-by-step explanation:

a.

(1.36 L)/(10 hr) = (0.136 L)/(hr)

Flow rate = (0.136 L)/(hr) × (1000 mL)/L = (136 mL)/(hr)

136 mL × (27 mg)/(100 mL) = 36.72 mg

Delivery rate = (36.72 mg)/(hr)

b.

(136 mL)/(hr) × (13 gtt)/(mL) = (1868 gtt)/(hr)

c.

10 hr × (36.72 mg)/)hr) = 367.2 mg

15. The area of a triangle is 72 in the base is 12 in. Find the height.​

Answers

Answer:

[tex]hright =12[/tex]

Step-by-step explanation:

----------------------------------------

The formula to find the area of a triangle is  [tex]A=\frac{1}{2}bh[/tex]  where [tex]b[/tex] stands for the base and [tex]h[/tex] stands for the height.

But we already know the area and the base. So to find the height, let's substitute 72 for [tex]A[/tex] and 12 for [tex]b[/tex], and solve.

[tex]72=\frac{1}{2}(12)(h)[/tex]

[tex]72=6h[/tex]

Here, divide both sides by 6

[tex]12=h[/tex]

--------------------

Hope this is helpful.

Answer:

height = 12

Step-by-step explanation:

.............

SOMEONE HELP ASAP PLES NO EXPLANATOIN NEEDED PLS LEAVE UR ANSWER AS TEXT (SOME TIMES I CAN'T SEE IMAGES) THANK YOU SO MUCH!!!

Answers

Answer:

i cant see the image

Step-by-step explanation:

At one point in history, the NBA finals required that one of the two teams win at least three of five games in order to win the Championship. If one team wins the first two games, what is the probability that the same team wins the Championship, assuming that the two teams are well matched and each team is equally likely to win each game

Answers

Answer:

50% i believe

Step-by-step explanation:

because in every scenario theres 2 teams and if they are well matched it be half and half on every game assuming they're the same level of comp

Please HELP!

How many pairs (A, B) are there where A and B are subsets of {1, 2, 3, 4, 5, 6, 7, 8} and A ∩ B has exactly two elements?

Answers

Answer:

There are 256 pairs in all.

Write the range of the function using interval notation.

Answers

Given:

The graph of a function.

To find:

The range of the given function using interval notation.

Solution:

Range: The set of y-values or output values are known as range.

From the given graph, it is clear that the function is defined for [tex]0<x<4[/tex] and the values of the functions lie between -2 and 2, where -2 is excluded and 2 is included.

Range [tex]=\{y|-2<y\leq 2\}[/tex]

The interval notation is:

Range [tex]=(-2,2][/tex]

Therefore, the range of the given function is (-2,2].

How would 0.42 be shown as a percent?
A. 0.42%
B. 4%
C. 4.2%
D. 42%

Answers

Answer:

42%

Step-by-step explanation:

to find percentages, you move the decimal point twice to the right

Last softball season, Pamela had 46 hits, a combination of singles (1 base), doubles (2 bases), and triples (3 bases). These 46 hits totaled 66 bases, and she had 4 times as many singles as doubles. How many doubles did she have?

Answers

Answer:

She had 8 doubles.

Step-by-step explanation:

This question is solved by a system of equations.

I am going to say that:

x is the number of singles.

y is the number of doubles

z is the number of triples.

46 hits

This means that [tex]x + y + z = 46[/tex]

46 hits totaled 66 bases

This means that:

[tex]x + 2y + 3z = 66[/tex]

4 times as many singles as doubles

This means that [tex]x = 4y[/tex]

So

[tex]x + 2y + 3z = 66[/tex]

[tex]4y + 2y + 3z = 66[/tex]

[tex]6y + 3z = 66[/tex]

And

[tex]x + y + z = 46[/tex]

[tex]4y + y + z = 46[/tex]

[tex]5y + z = 46 \rightarrow z = 46 - 5y[/tex]

Then

[tex]6y + 3z = 66[/tex]

[tex]6y + 3(46 - 5y) = 66[/tex]

[tex]6y + 138 - 15y = 66[/tex]

[tex]9y = 72[/tex]

[tex]y = \frac{72}{9}[/tex]

[tex]y = 8[/tex]

She had 8 doubles.

The diagram shows triangle ABC.
С
Work out the sizes of angles x, y and z.
40°
110°
х
Z
A
В

Answers

Answer:

x=70

y=30

z=20

Step-by-step explanation:

x=180-110 (angles on a straight line)

y=180-110-40 (angle sum of triangle)

z= 180-90-70 (angle sum of triangle)

Answer:

x=70°

y=30°

z=20°

Step-by-step explanation:

x=180°-110°(anlges on a straight line)

x=70°

y+110°+40°=180°(sum of angles of triangle)

y+150°=180°

y=180°-150°

y=30°

z+x+90°=180°(sum of angles of triangle)

z+70°+90°=180°

z+160°=180°

z=180°-160°

z=20°

x(x-y) - y( x- y) simplify ​

Answers

Step-by-step explanation:

x²-xy-xy+y²

x²+2xy+y²

hope it helps

Let f(x)
2x + 8, g(x) = x2 + 2x – 8, and h(x) = 3x – 6.
Perform the indicated operation. (Simplify as far as possible.)
(h · f)(3) =

Answers

Answer:

36

Step-by-step explanation:

(h · f)(x) = h(f(x))

h(f(x)) = h(2x+8)

h(f(x))= 3(2x+8) - 6

h(f(x)) = 6x + 24 - 6

h(f(x))= 6x + 18

If x = 3

h(f(x))= 6(3) + 18

h(f(x))= 18 + 18

h(f(x))= 36

Hence (h · f)(3) = 36

Find the range of the data.
Scores: 81, 79, 80, 88, 72, 96, 86, 73, 79, 88

Answers

Answer:

24

Step-by-step explanation:

To find the range, you must subtract the lowest value from the highest value in the data set. If you organize the set from least to greatest, 72 is the lowest, and 96 is the highest.

So, 96 - 72 = 24, which is the range.

The function f is defined by the following rule. f(x) = 5x+1 Complete the function table.

Answers

Answer:

[tex]-5 \to -24[/tex]

[tex]-1 \to -4[/tex]

[tex]2 \to 11[/tex]

[tex]3 \to 16[/tex]

[tex]4 \to 21[/tex]

Step-by-step explanation:

Given

[tex]f(x) = 5x + 1[/tex]

Required

Complete the table (see attachment)

When x = -5

[tex]f(-5) = 5 * -5 + 1 = -24[/tex]

When x = -1

[tex]f(-1) = 5 * -1 + 1 = -4[/tex]

When x = 2

[tex]f(2) = 5 * 2 + 1 = 11[/tex]

When x = 3

[tex]f(3) = 5 * 3 + 1 = 16[/tex]

When x = 4

[tex]f(4) = 5 * 4 + 1 = 21[/tex]

So, the table is:

[tex]-5 \to -24[/tex]

[tex]-1 \to -4[/tex]

[tex]2 \to 11[/tex]

[tex]3 \to 16[/tex]

[tex]4 \to 21[/tex]

The sum of the first ten terms of an arithmetic progression consisting of

positive integer terms is equal to the sum of the 20th, 21st and 22nd term.

If the first term is less than 20, find how many terms are required to give

a sum of 960.

Answers

Answer: [tex]n=13[/tex]

Step-by-step explanation:

Given

Sum of the first 10 terms is equal to sum of 20, 21, and 22 term

[tex]\Rightarrow \dfrac{10}{2}[2a+(10-1)d]=[a+19d]+[a+20d]+[a+21d]\\\\\Rightarrow 5[2a+9d]=3a+60d\\\Rightarrow 10a+45d=3a+60d\\\Rightarrow 7a=15d[/tex]

No of terms to give a sum of 960

[tex]\Rightarrow 960=\dfrac{n}{2}[2a+(n-1)d]\\\\\Rightarrow 1920=n[2a+(n-1)\cdot \dfrac{7}{15}a]\\\\\Rightarrow 28,800=n[30a+7a(n-1)]\\\\\Rightarrow a=\dfrac{28,800}{n[30+7n-7]}\\\\\Rightarrow a=\dfrac{28,800}{n[23+7n]}[/tex]

Value of first term is less than 20

[tex]\therefore \dfrac{28,800}{n[23+7n]}<20\\\\\Rightarrow 28,800<20n[23+7n]\\\Rightarrow 0<460n+140n^2-28,800\\\Rightarrow 140n^2+460n-28,800>0\\\\\Rightarrow n>12.79\\\\\text{For integer value }\\\Rightarrow n=13[/tex]

Answer:

15

Step-by-step explanation:

In the previous answer halfway through they used the equation: 960 = (n÷2)×(2a+(n-1)×(7a÷15))

Using this equation we can substitute an number to replace n, the higher the number is the smaller a would be.

When we substitute 15 into a, then it leaves us with the answer to be a = 15 which is a positive integer and also is smaller than 20, this then let’s us know that 15 is how many terms can be summed up to make 960.

To double check this answer you can find that d = 7 by changing the a into 15 in the formula 7a/15 (found in the previous answer.

Then in the expression: (n÷2)×(2a+(n-1)×d)

substitute:

n = 14 (must be an even number for the equation to work)

a = 15

d = 7

This will give you an answer of 847, but this is only 14 terms as we changed n into 14. To add the final term you need to complete the following equation: 847+(a+(n-1)×d)

substituting:

n = 15

a = 15

d = 7

This will give you the answer of 960, again proving that it takes 15 terms to sum together to make the number 960.

I hope this has helped you.

P.S. Everything in the previous solution was right apart from the start of the last section and the answer

which of the following function shows the absolute value parent function FX=lxl shifted up

Answers

Answer:

The answer is C.

as for C . the value of f(x) increases by 7 and so the graph goes up by units 7.

OR

g(x) = |x| + 7

we know that |x| is f(x), so :-

g(x) = f(x) + 7

and since f(x) is plot on y- axis the graph climbs the y axis by 7 units

*The graph shifts right or left for the other functions*

Meghan sells advertisements for a radio station. Each 30 second ad costs $20 per play, and each 60 second ad
costs $35 per play. Meghan sold 12 ads for $315. She wrote the system below letting x represent the number of 30
second ads and y represent the number of 60 second ads.
X+ y = 12
20x+35y = 315
What is the solution to the system of equations?

Need answers ASAP!!!!

Answers

Answer:

usai964s46s694s4o6s64694s946649s469 opps

Answer:

[tex](x,y)=(7,5)[/tex]

Step-by-step explanation:

Megan's equation will be:

[tex]20x+35y=315[/tex]

[tex]x+y=12[/tex]

Substitute [tex]x=12-y[/tex] in the first equation:

[tex]20(12-y)+35y=315[/tex]

[tex]15y=75[/tex]

[tex]y=75/15[/tex]

[tex]y=5[/tex]

Find x:

[tex]x=12-5[/tex]

[tex]x=7[/tex]

Where x and y represent 30-second and 60-second ads sold, we find that Meghan's sales were:

[tex](x,y)=(7,5)[/tex]

hope this helps....

For each of the following, assume that the two samples are obtained from populations with the same mean, and calculate how much difference should be expected, on average, between the two sample means. Each sample has n =4 scores with s^2 = 68 for the first sample and s^2 = 76 for the second. (Note: Because the two samples are the same size, the pooled variance is equal to the average of the two sample variances).
a) 4.24.
b) 0.24.
c) 8.48.
d) 6.00.
Next, each sample has n=16 scores with s^2 = 68 for the first sample and s^2 = 76 for the second.
a) 0.12.
b) 2.12.
c) 4.24.
d) 3.00.

Answers

Answer:

d)6.00

d)3.00

Step-by-step explanation:

We are given that

n=4 scores

[tex]S^2_1=68[/tex]

[tex]S^2_2=76[/tex]

We have to find the  difference should be expected, on average, between the two sample means.

[tex]S_{M_1-M_2}=\sqrt{\frac{S^2_1}{n_1}+\frac{S^2_2}{n_2}}[/tex]

[tex]n_1=n_2=4[/tex]

Using the formula

[tex]S_{M_1-M_2}=\sqrt{\frac{68}{4}+\frac{76}{4}}[/tex]

[tex]S_{M_1-M_2}=\sqrt{\frac{68+76}{4}}[/tex]

[tex]S_{M_1-M_2}=\sqrt{36}=6[/tex]

Option d is correct.

Now, replace n by 16

[tex]n_1=n_2=16[/tex]

[tex]S_{M_1-M_2}=\sqrt{\frac{68}{16}+\frac{76}{16}}[/tex]

[tex]S_{M_1-M_2}=\sqrt{\frac{68+76}{16}}[/tex]

[tex]S_{M_1-M_2}=\sqrt{9}=3[/tex]

Option d is correct.

Find the solution of the differential equation that satisfies the given initial condition. (dP)/(dt)

Answers

Answer:

[tex]P = (\frac{1}{3}t^\frac{3}{2} + \sqrt 2 - \frac{1}{3})^2[/tex]

Step-by-step explanation:

Given

[tex]\frac{dP}{dt} = \sqrt{Pt[/tex]

[tex]P(1) = 2[/tex]

Required

The solution

We have:

[tex]\frac{dP}{dt} = \sqrt{Pt[/tex]

[tex]\frac{dP}{dt} = (Pt)^\frac{1}{2}[/tex]

Split

[tex]\frac{dP}{dt} = P^\frac{1}{2} * t^\frac{1}{2}[/tex]

Divide both sides by [tex]P^\frac{1}{2}[/tex]

[tex]\frac{dP}{ P^\frac{1}{2}*dt} = t^\frac{1}{2}[/tex]

Multiply both sides by dt

[tex]\frac{dP}{ P^\frac{1}{2}} = t^\frac{1}{2} \cdot dt[/tex]

Integrate

[tex]\int \frac{dP}{ P^\frac{1}{2}} = \int t^\frac{1}{2} \cdot dt[/tex]

Rewrite as:

[tex]\int dP \cdot P^\frac{-1}{2} = \int t^\frac{1}{2} \cdot dt[/tex]

Integrate the left hand side

[tex]\frac{P^{\frac{-1}{2}+1}}{\frac{-1}{2}+1} = \int t^\frac{1}{2} \cdot dt[/tex]

[tex]\frac{P^{\frac{-1}{2}+1}}{\frac{1}{2}} = \int t^\frac{1}{2} \cdot dt[/tex]

[tex]2P^{\frac{1}{2}} = \int t^\frac{1}{2} \cdot dt[/tex]

Integrate the right hand side

[tex]2P^{\frac{1}{2}} = \frac{t^{\frac{1}{2} +1 }}{\frac{1}{2} +1 } + c[/tex]

[tex]2P^{\frac{1}{2}} = \frac{t^{\frac{3}{2}}}{\frac{3}{2} } + c[/tex]

[tex]2P^{\frac{1}{2}} = \frac{2}{3}t^\frac{3}{2} + c[/tex] ---- (1)

To solve for c, we first make c the subject

[tex]c = 2P^{\frac{1}{2}} - \frac{2}{3}t^\frac{3}{2}[/tex]

[tex]P(1) = 2[/tex] means

[tex]t = 1; P =2[/tex]

So:

[tex]c = 2*2^{\frac{1}{2}} - \frac{2}{3}*1^\frac{3}{2}[/tex]

[tex]c = 2*2^{\frac{1}{2}} - \frac{2}{3}*1[/tex]

[tex]c = 2\sqrt 2 - \frac{2}{3}[/tex]

So, we have:

[tex]2P^{\frac{1}{2}} = \frac{2}{3}t^\frac{3}{2} + c[/tex]

[tex]2P^{\frac{1}{2}} = \frac{2}{3}t^\frac{3}{2} + 2\sqrt 2 - \frac{2}{3}[/tex]

Divide through by 2

[tex]P^{\frac{1}{2}} = \frac{1}{3}t^\frac{3}{2} + \sqrt 2 - \frac{1}{3}[/tex]

Square both sides

[tex]P = (\frac{1}{3}t^\frac{3}{2} + \sqrt 2 - \frac{1}{3})^2[/tex]

HELP
-5(2m-3)-4<81
I need the steps also well

Answers

Answer:

m>-7

Step-by-step explanation:

expand

-10m+15-4<81

-10m+11<81

collect like terms

-10m<81-11

-10m<70

m>-7

Suppose that you are thinking about buying a car and have narrowed down your choices to two options.
The new-car option: The new car costs $25,000 and can be financed with a four-year loan at 6.12%.
The used-car option: A three-year old model of the same car costs $17,000 and can be financed with a three-year loan at 7.72%.
=||)
[1-(2-4) 11
What is the difference in monthly payments between financing the new car and financing the used car? Use PMT
The difference in monthly payments between financing the new car and financing the used car is $
(Round to the nearest cent as needed.)

Answers

Answer:

sjsjsuduhr r ki snsbtsuwi 3 38yv4r djvs

A sample of 25 one-year-old girls had a mean weight of 24.1 pounds with a standard deviation of pounds. Assume that the population of weights is normally distributed. A pediatrician claims that the standard deviation of the weights of one-year-old girls is less than pounds. Do the data provide convincing evidence that the pediatrician's claim is true

Answers

Answer:

Paedtricians claim isn't true.

Step-by-step explanation:

The hypothesis :

H0 : σ = 7

H0 : σ > 7

The test statistic ; χ² :

χ² = [(n - 1) * s²] ÷ σ²

n = 25 ; s = 4.3, σ = 7

χ² = [(25 - 1) * 4.3²] ÷ 7²

χ² = [(24 * 4.3²] ÷ 49

χ² = 443.76 / 49

χ² = 9.056

At α = 0.01 ; critical value = 42.980

Since critical value > test statistic, we fail to reject the null, H0.

Private nonprofit four-year colleges charge, on average, $26,208 per year in tuition and fees. The standard deviation is $7,040. Assume the distribution is normal. Let X be the cost for a randomly selected college. Round all answers to 4 decimal places where possible.

a. What is the distribution of X? X ~ N(
26208
Correct,
7040
Correct)

b. Find the probability that a randomly selected Private nonprofit four-year college will cost less than 22,924 per year.


c. Find the 60th percentile for this distribution. $
(Round to the nearest dollar.)

Answers

Answer:

#########

Step-by-step explanation:

Which simplified fraction is equal to 0.53? Need answers now plz

Answers

Answer:

8/15

Step-by-step explanation:

Answer:

8/15

Step-by-step explanation:

when you divide 8/15 its 0.53

The parametric equations for the paths of two projectiles are given. At what rate is the distance between the two objects changing at the given value of t? (Round your answer to two decimal places.) x1 = 10 cos(2t), y1 = 6 sin(2t) First object x2 = 4 cos(t), y2 = 4 sin(t) Second object t = π/2

Answers

Answer:

-  [tex]\frac{4}{\sqrt{29} }[/tex]

Step-by-step explanation:

The equations for the 1st object :

x₁ = 10 cos(2t),  and  y₁ = 6 sin(2t)

2nd object :

x₂ = 4 cos(t), y₂ = 4 sin(t)

Determine rate at which distance between objects will continue to change

solution Attached below

Distance( D )  = [tex]\sqrt{(10cos2(t) - 4cos(t))^2 + (6sin2(t) -4sin(t))^2}[/tex]

hence; dD/dt = - [tex]\frac{4}{\sqrt{29} }[/tex]

A school contains 140 boys and 160 girls. what is the ratio of boys to girls?
I need full working out please

Answers

Answer:

7 : 8

Step-by-step explanation:

that is the procedure above

Other Questions
What is the word form of 1.604 Using the net below, find the surface areaof the pyramid.sto5 in5 in.Surface Area=[?] in?Enter What is the solution to the equation below?2x/x-1=2 A. 4 B. 2 C. 5 D.3 An electron is pushed into an electric field where it acquires a 1-V electrical potential. Suppose instead that two electrons are pushed the same distance into the same electric field (but far enough apart that they don't effect eachother). What is the electrical potential of one of the electrons now? when a company following a differentiation strategy ensures that the higher price it charges for its higher quality is not priced too far about the price of competition, the company is using the process of Question 3: use the image and your knowledge of the isosceles triangle to find the value of x que elementos hacen que el texto de las tres tazas de vergara no sea romantico What are the steps in order of operations Read the excerpt from Part 2 of The Odyssey.Neither reply nor pity came from him,but in one stride he clutched at my companionsand caught two in his hands like squirming puppiesto beat their brains out, spattering the floor.What does the simile add to the passage? Check all that apply.a description of the Cyclops homea comparison between Odysseus and his menan image of helplessnessa visual image of the Cyclops petsa comparison between the men and innocent animals Cyclical unemployment arises when:______. a. the agriculture sector completes the cycle of planting, cultivating, and harvesting the nation's food supply. b. labor unions strike for higher wages. c. the business cycle enters an expansionary phase. d. business activity in the macroeconomy declines. What do you think a child most understand to study cultural diversity? Present simple You usually up at Go'clock, mum You should avoid having other people in portraits with children. True False Determine which of the following statements is true about energy in ecosystems A. Consumers pass on 20% of the energy they get from a lower trophic level. OB. Primary consumers lose 60% of the energy they absorb from producers as urine and feces. C. 10% of the energy Primary consumers absorb is lost as heat. Ons Primary producers lose 20% of the energy they absorb as urine and feces. Hi, I'm new, please can you help me with these math exercises Thank you very much:3 given the algebraic fraction p (x) 2x2-2x determine its numerical value for x: 3 3x-1 given the algebraic fraction p (xy) - 3xy-y2 + x xy- + determine its numerical value for x- {3 y- {2 p (x, y) part 2: perform the following exercises by factoring a) b8 p4 + b: up3 -12 b7 p2 + 15u2 + b3 p5: b ) 32 p2 -24 p3 + 16 p2 -8 p4: part 3: perform the following equations a) 4x + 5 + 2: 3x-2 + 5b) 2-9x + 6: 2x-4x-6c) 3x-5 + 2: 2 + 4x-3d) bx-5x + 4: 3x + 5x + 2thank you A cylinder has a radius of 2.5 inches (in.) and a height of 11 in., as shown.2.5 in.11 in.What is the surface area, in square inches, of the cylinder? (iii) If a, b, c are rational numbers, thena x (b-c) #ax b-ax c. true or false write a letter to ur father friend expressing ur wish and requesting him to appeal to ur father to let u study the subject of ur choice Find the length of the line AB A salesman receives a salary of 250 a month. To this is added 5/4% of the sales that he makes. what is his pay for a month in which he makes 1500 in sales.